Si Fermat avait la solution.. - Page 2
Répondre à la discussion
Page 2 sur 3 PremièrePremière 2 DernièreDernière
Affichage des résultats 31 à 60 sur 83

Si Fermat avait la solution..



  1. #31
    invite7863222222222
    Invité

    Re : Si Fermat avait la solution..


    ------

    Leg je propose que tu écrives une et une seule phrase mathématique et que les mathématiciens de ce forum te disent si oui ou non ils la comprennent.

    Il faut bien partir d'un début pour pouvoir avancer.

    moi aussi ca commence à me frustrer et je ne suis meme pas mathématicien. Alors allez-y mais une seule phrase silvousplait.

    -----

  2. #32
    leg

    Re : Si Fermat avait la solution..

    bonne lecture et bon dimanche à lundi

    leg

  3. #33
    ericcc

    Re : Si Fermat avait la solution..

    Pour avoir déjà vu des discussions de ce type sur un fil précédent, j'ai le sentiment que Leg démontre qu'un triplet ne peut à la fois vérifier x²+y²=z² et l'équation
    de Fermat à un degré plus élevé.
    Mais peut être ai-je mal compris ?

  4. #34
    erik

    Re : Si Fermat avait la solution..

    Mais peut être ai-je mal compris ?
    Je ne crois pas que tu ais mal compris, effectivement leg démontre que
    si x²+y²=z² alors on ne peut pas avoir x^n+y^n=z^n

    Il reste donc à démontrer que même lorsque x²+y² différent de z² on ne peux toujours pas avoir x^n+y^n=z^n.

    Ce que ne fait pas leg (d'ailleur il me semble qu'il n'avait pas compris que cette étape était nécessaire pour que la démonstration soit complète)

  5. #35
    leg

    Re : Si Fermat avait la solution..

    Citation Envoyé par erik Voir le message
    Je ne crois pas que tu ais mal compris, effectivement leg démontre que
    si x²+y²=z² alors on ne peut pas avoir x^n+y^n=z^n

    Il reste donc à démontrer que même lorsque x²+y² différent de z² on ne peux toujours pas avoir x^n+y^n=z^n.

    Ce que ne fait pas leg (d'ailleur il me semble qu'il n'avait pas compris que cette étape était nécessaire pour que la démonstration soit complète)
    bonjour
    x^n+y^n=z^n. si cette équation est vrait comment l'écrivez vous en utilisant la formule des triplets Pyth
    si vous avez bien regardé le résumé de samedi
    c'est biern sous cette forme:
    (p²-q²)² +(2pq)² = (p²+q²)² pour N >2
    de plus il est évident que x² + y² = z² ne peut être égale à x^n +y^n = z^n, pour n premier > 2;

    mais sans perte de généralité ces deux équation s'écrive de la même façon! c'est à dire de cette forme:
    (p²-q²)² +(2pq)² = (p²+q²)²
    non !
    ce qui implique que quelque soit le triplet de racines carrées il est de la forme p²-q², 2pq, p²+q²!

    soit on a des racines carrées entières, ce qui veut bien dire : des carrés, ou des cubes , ou des entiers à la puissance 5....etc etc

    soit ces racines carrées sont algébriques pour les puissance N premier en commençant par N=3 ,
    mais elles sont toujours de la forme de p²-q²,2pq,p²+q²! et il est tout aussi évident que dans ce cas, p et q ne sont plus des entiers mais des réels, algébriques ou non!

    or si p et q sont choisis dans les racines carrées algébriques des entiers à la puissanc ,par exemple N =3 :

    que sont p et q mis au carré?

    a vous

  6. #36
    leg

    Re : Si Fermat avait la solution..

    encore une autre question:
    est ce qu'au moins, vous concevez, que quelque soit la solution qui existerait, dans une puissance première N >2
    ou dans le double de cette puissance première c'est à dire 6,10,14 et qui dans ce cas est aussi solution dans N =2 mais aussi dans une de ces puissance première >2;

    elle serait alors de la forme (p²-q²)²+(2pq)²=(p²+q²)² ?

    avec p et q entiers pour une solution dans les puissances ,6,10,14..etc
    et bien évidement non entiers pour N = 3,5,7..etc

    donc regardez bien le cas N=4 et la contradiction qui en ressort , sur le résumé de samedi 14h
    A vous, merci

  7. #37
    Médiat

    Re : Si Fermat avait la solution..

    Citation Envoyé par leg Voir le message
    mais sans perte de généralité ces deux équation [ x² + y² = z² et x^n +y^n = z^n] s'écrive de la même façon! c'est à dire de cette forme:
    (p²-q²)² +(2pq)² = (p²+q²)²
    non !
    Je ne sais, pas mais j'aimerais bien en voir une démonstration lisible par un pauvre mathématicien comme moi...
    Je suis Charlie.
    J'affirme péremptoirement que toute affirmation péremptoire est fausse

  8. #38
    leg

    Re : Si Fermat avait la solution..

    Citation Envoyé par Médiat Voir le message
    Je ne sais, pas mais j'aimerais bien en voir une démonstration lisible par un pauvre mathématicien comme moi...
    si je comprend bien ta question Mediat tu supposerais qu'il n'est pas possible d'écrire :
    [ x² + y² = z² et x^n +y^n = z^n] s'écrive de la même façon! c'est à dire de cette forme:
    (p²-q²)² +(2pq)² = (p²+q²)²

    c'est à dire sous la forme utilisant la formule des triangle rectangle , donc supposons que tu est raison , il est impossible qu'il existe un triangle rectangle de réel ! qui plus est, par exemple en mettant les côtés au carré je ne pourrai vérifier le théorème de pythagore. ex 3 + 1 = 4
    pour cette solution j'ai donné les valeur de p' et q'

    donc,plus simple: p = sqrt 2 et q = sqrt de 3 ou l'inverse, peu importe
    tel que: q² = 2 et p² = 3
    soit p² + q² = 5
    p²-q² = 1
    2pq = sqrt de 24
    soit la solution 1 + 24 = 25 où 25 est bien le carré de lhypoténuse et 1 est 24 sont les carrés des côtés de ce triangle rectangle !

    ce qui à déja été débatu sur ce forum sans contradiction

    de plus
    X tel que X = p²-q² = (p-q²) + ((p-q) n) avec n =2 pour q =1
    donc cette formule permet de reconstituer les deux paramètres p et q , quelqu'il soit ;ce que tu peux vérifier. il en existe même une autre formule .
    A vous

  9. #39
    Médiat

    Re : Si Fermat avait la solution..

    Désolé, mais je ne vois toujours pas de démonstration que si :

    x0^3 + y0^3 = z0^3,
    alors il existe p et q tels que
    x0 = (p²-q²)
    y0 = (2pq)
    z0 = (p²+q²)

    (x0, y0, z0 sont des entiers non nuls ; je prends l'équation de Fermat pour n = 3, pour simplifier et utiliser un cas où la démonstration est connue depuis longtemps)
    Je suis Charlie.
    J'affirme péremptoirement que toute affirmation péremptoire est fausse

  10. #40
    leg

    Re : Si Fermat avait la solution..

    Médiat
    il est clair que si il était impossible d'écrire ces équations sous la forme cité, alors Fermat ne risquait pas de s'avancer à dire qu'il avait une démo merveilleuse mais de plus le cas n=4 tel que z et y soit carré dans un tiplet serait faux, ainsi d'ailleur que X 'et Z carré dans un triplet car comment écrire le premier triplet 3,4 et 5
    tel x =3,y = 4 et z =5 où :
    x² = (z+y) (z-y)
    c'est à dire que z et y sont premiers entre eux, donc les deux nombres (z+y) et (z-y) le sont aussi deux à deux et comme leur produit est un carré, ces deux nombres le sont aussi
    et bien évidement: (z+y) = u² ainsi que (z-y) = v²
    alors il existe p' et q' < p et q
    tel que :
    (p'² - q'²)² + (2p'q')² = u² = (p'² + q'²) où bien évidement p' et q' ne peuvent être des entier pour cet exmple mais aussi pour (z-y) = v² former par p'' et q''
    tel que:
    (p''² + q''²) - (2p''q'')² = v² = (p''² + q''²),

    il est trivial de constater que (p'² - q'²)² = (p''² + q''²)
    mais imaginez avec des entiers, des carrés ou des cubes ..etc ..etc

  11. #41
    leg

    Re : Si Fermat avait la solution..

    Citation Envoyé par Médiat Voir le message
    Désolé, mais je ne vois toujours pas de démonstration que si :

    x0^3 + y0^3 = z0^3,
    alors il existe p et q tels que
    x0 = (p²-q²)
    y0 = (2pq)
    z0 = (p²+q²)

    (x0, y0, z0 sont des entiers non nuls ; je prends l'équation de Fermat pour n = 3, pour simplifier et utiliser un cas où la démonstration est connue depuis longtemps)
    ok je comprend

    je ne l'ai effectivement pas indiquée car il me semblait que c'était évident du fait que x0 ,y0 et z0 forme obligatoirement un triangle rectangle si et seulement si
    x0^3 + y0^3 = z0^3, existe!
    par exemple en formant un carré de côté A = x0+y0 et un carré interieur de côté z0 tel que :
    A² = (x0² +y0²) + (2 x0 y0)
    ou
    A² = z0² + (2 x0 y0)
    en suprimant les deux (2 x0 y0)

    je pensais qu'il resterait z0² = x0² +y0²
    donc ,il existe forcément p et q tels que
    x0 = (p²-q²)
    y0 = (2pq)
    z0 = (p²+q²)
    car c'est le principe même du théorème de pythagore.

    voila;
    Mais si c'est faux alors effectivementje ne pourrai avoir reconstruit ce qui je pense était l'idée de Fermat, mais je pense qu'il vous est facile de montrer, aussi que ce simple cas est vrai d'une autre façon..
    amicalement

  12. #42
    leg

    Re : Si Fermat avait la solution..

    Médiat
    je n'arrive pas comprendre comment tu peux supposer, ta question sur ton post #39
    car enfin si ton triplet d'entiers non nul x0, y0 , z0 existe tel qu'il implique x0^3 + y0^3 = z0^3,
    p² - q² = sqrt de x0^3 et non pas x0
    2pq = sqrt de y0^3 et non pas yo
    p²+q² = sqrt de z0^3, et non pas z0

    mais par contre si:
    il existe p et q tels que
    x0 = (p²-q²)
    y0 = (2pq)
    z0 = (p²+q²)
    alors la solution est de la forme :
    x0^3 + y0^3 = z0^3, si est seulement si
    x0,y0,z0 sont trois cubes ! ce qui impliquerai aussi une solution dans la puissance 6 et 2

    si p²-q² est un carré (x²) ou dire que p²-q² = x0 un cube, aucune contradiction;

    mais cela ne veut pas dire que x² = x0 ; on est bien ok

  13. #43
    ericcc

    Re : Si Fermat avait la solution..

    Mediat :"Tu vas à la pêche ?"
    Leg : "Non je vais à la pêche"
    Mediat : "Ah bon, je croyais que tu allais à la pêche"

  14. #44
    Médiat

    Re : Si Fermat avait la solution..

    Citation Envoyé par leg Voir le message
    je ne l'ai effectivement pas indiquée car il me semblait que c'était évident du fait que x0 ,y0 et z0 forme obligatoirement un triangle rectangle si et seulement si x0^3 + y0^3 = z0^3, existe!
    Voila bien le genre d'évidence que je ne trouve pas ... évidente, mais plutôt fausse
    1^3 + 1^3 = RacineCubique(2)^3, alors que des côtés de longueur 1, 1 et RacineCubique(2) ne forment pas un triangle rectangle.
    Je suis Charlie.
    J'affirme péremptoirement que toute affirmation péremptoire est fausse

  15. #45
    leg

    Re : Si Fermat avait la solution..

    Enfin Médiat tu le fais exprés
    depuis que l'on parle de ces triplet p et q >0 avec p>q ou q>p
    de plus x0,y0 et z0
    sont tel que x0<yo ou l'inverse alors ne fait pas dans l'ignorance
    car ton post si il sous entendait 1^3 +1^3 =z^3 tu aurait pu tout aussi bien dire que x=y alors que je ne parle depuis le début que de racines carrées d'entiers à la puissance N < ou = 2; et je ne vois pas dans ce cas; ce que vient faire cette allusion
    1^3 +1^3 =z^3
    ce qui justifie bien le propos d'ericc

    au risque de te décevoir
    voici ton post:
    Désolé, mais je ne vois toujours pas de démonstration que si :

    x0^3 + y0^3 = z0^3,
    alors il existe p et q tels que

    x0 = (p²-q²)
    y0 = (2pq)
    z0 = (p²+q²)

    ce que je n'ai jamais dit!

    puisque ce sont des entiers non nul je te dit donc que : x0,y0 et z0 sont trois carrés parfait, ce qui justifie
    x0^3 + y0^3 = z0^3, d'où il existe
    a0 = (p²-q²)
    b0 = (2pq)
    c0 = (p²+q²)
    trois cubes qui justifient aussi : x0^3 + y0^3 = z0^3
    si et seulement si il existerait une solution dans ces puissances 3 ,6 et 2

  16. #46
    leg

    Re : Si Fermat avait la solution..

    que sous entendait Fermat en disant dans ces courriers et dans son diophante
    si on donne deux carrée il en existerait deux plus petits qui aurait la même propriété ..ect et deux façon générale il en serait de même pour toute puissance n >2
    c'est tout simplement une généralisation du cas N = 4

  17. #47
    ericcc

    Re : Si Fermat avait la solution..



    C'est bien ce que je disais plus haut : leg montre (enfin je n'ai pas vraiment été au bout de sa démonstration quelque peu chantournée...) qu'un triplet "pythagoricien" ne peut en même temps être solution de l'équation de Fermat pour N>2.


  18. #48
    Médiat

    Re : Si Fermat avait la solution..

    Dernier essai :

    Peux-tu démontrer la phrase suivante extraite de ton post N°41, en explicitant le sens du verbe "existe" dans celle-ci, car son sujet semble être une équation, ce qui me perturbe un peu :

    il me semblait que c'était évident du fait que x0 ,y0 et z0 forme obligatoirement un triangle rectangle si et seulement si x0^3 + y0^3 = z0^3, existe
    Je suis Charlie.
    J'affirme péremptoirement que toute affirmation péremptoire est fausse

  19. #49
    leg

    Re : Si Fermat avait la solution..

    Citation Envoyé par ericcc Voir le message


    C'est bien ce que je disais plus haut : leg montre (enfin je n'ai pas vraiment été au bout de sa démonstration quelque peu chantournée...) qu'un triplet "pythagoricien" ne peut en même temps être solution de l'équation de Fermat pour N>2.

    c'est presque çà pour N pair > 2, c'est exact!
    pour N premier > 2 le triplet pythagoricien n'est pas constitué d'entier, mais j'utilise la même méthode et je me sert du de la contradiction du cas n=4 puis N=6 pour faire remarquer qu'il est impossible de paramètrer a juste titre, un triplet pythagoricien de racines carrées (x',y' z') d'entiers à la puissance N = 3 et non N=6

  20. #50
    erik

    Re : Si Fermat avait la solution..

    C'est bien ce que je disais plus haut : leg montre (enfin je n'ai pas vraiment été au bout de sa démonstration quelque peu chantournée...) qu'un triplet "pythagoricien" ne peut en même temps être solution de l'équation de Fermat pour N>2.
    Et oui, et ça ce n'est pas le théorème de Fermat, c'est juste le théorème de Leg (qui est moins général que celui de Fermat)

  21. #51
    leg

    Re : Si Fermat avait la solution..

    Citation Envoyé par Médiat Voir le message
    Dernier essai :

    Peux-tu démontrer la phrase suivante extraite de ton post N°41, en explicitant le sens du verbe "existe" dans celle-ci, car son sujet semble être une équation, ce qui me perturbe un peu :
    en explicitant le sens du verbe "existe" :

    tout simplement si cette solution ou cette équation existe c'est à dire: que je suppose que x0^3 + y0^3 = z0^3, est vrai!

    tu me demande de redémontrer que c'est impossible ?

  22. #52
    leg

    Re : Si Fermat avait la solution..

    suppose que cette solution soit vraie:
    x0^3 + y0^3 = z0^3,
    il existe deux cas possibles
    1)
    dans N =3 avec solutrion aussi dans N=6 et 2:

    x0,y0 et z0 sont trois entiers non nuls premiers entre eux ;tel que x0<y0 <z0 ou y0<x0 <z0
    alors x0,y0 z0 sont trois carrés parfaits il existe alors deux entiers non nul p et q premier entre eux p > q
    tel que :
    p² -q² = a^3 , 2pq = b^3 , p²+q²= c^3
    d'où: la solution x0^3 + y0^3 = z0^3 équivalent a :
    (a^3)² + (b^3)² = (c^3)²
    soit l'égalité suivante
    p² -q²)² + (2pq)² =(p²+q²)²

    mais si (a^3)² est vrai alors ((c^3) +(b^3)) ((c^3) -(b^3)) est vrai !
    donc comme c'est deux nombres sont aussi premiers entre eux deux à deux, leur produit est un cube au carré!
    puisque (a^3)² est vrai!
    ce qui m'a bien redonné deux solutions cubiques par addition et soutraction ce qui est impossible!
    mais ces deux solutions ne peuvent pas avoir été paramètré par un couple d'entiers p' et q' avec p'< p et q'< q ; si ces deux solutions existaient, elles seraient paramétraient par deux couples de réels p'' et q'' ainsi que p''' et q'''
    la même contradiction que le cas N=4
    comme il n'existe pas deux cubes qui peuvent donner un cube par addition et soustraction il ne peut donc exister deux couples de réel qui aurait paramétré ces deux solutions impossibles
    et en continuant si il existait des solution dans la puissance N=3 uniquement il me serait possible avec des réel de paramétrer ces deux solutions (impossible) et de trouver aussi une solution dans N=6!

    mais supposons qu'il exiterait quand même une solution dans N=3 mais pas dans N=6
    comme pour le cas N=2 il y a des solutions mais pas dans dans N=4 c'est à dire un triplet de carrés qui donnerait deux solutions par addition et soustraction! (mais obligatoirement paramètré par deus couple de réels
    p'' et q'' ainsi que p''' et q''' )


    donc deuxième cas
    2)la solution est vrai que pour N=3 : x0^3 + y0^3 = z0^3,

    il existerait p'' et q'' réel; mais en aucun cas algébrique !
    a)
    c'est à dire que je ne peux pas les choisir p'' et q''(comme pour N=2) dans les racines carrées de ces cubes (j'entends par là racines carrées d'entiers à la puissance 3)
    en effet en méttant p'' et q'' au carré pour obtenir x' qui est la racine carrée d'un cube, ainsi que pour z' qui est la racine carrée de l'hypoténuse d'un cube, j'obtien la contradiction cité au dessus, deux solutions cubiques par addition et soustraction

    b) je n'ai donc qu'un choix p'' et q'' réels, afin de paramètré le triplet de racines carrées de cubes:
    x',y' et z' qui mis au carré va me donner la solution :
    x0^3 + y0^3 = z0^3,
    or c'est là que je pouvais me servir de la démo de L Euler en supposant que Fermat l'ai aussi démontrée, il n'existe pas de solution cubique, c'est à dire qu'il n'existe pas non plus, deux réels p'' et q'' qui peuvent pramétrer un tel triplet de racine carrées de cubes

    donc Fermat aurait pu avoir eu besoin, si il le fallait d'une démonstration complémentaire uniquement pour le cas N=3 afin de généraliser!
    car pas de couple de réels pour N=3 4et 6 mais aussi pour N=2 du fait que toutes les solution entières sont données par p et q tel que défini, car le contraire indiquerait qu'il exsite des solutions entières et primitives qui ne sont données que par un couples de réel p'' et q'' ce qui est absurde! la formule des triplets PyT serait fausse!

    c'est inutile pourrait on dire en regardant ces cas,
    mais il y a encore des contradictions avec le couple de paramètres p'' et q'' si il existait! sans avoir recour à la démo de L Euler, qui permettent de conclure définitivement.

  23. #53
    leg

    Re : Si Fermat avait la solution..

    Citation Envoyé par erik Voir le message
    Et oui, et ça ce n'est pas le théorème de Fermat, c'est juste le théorème de Leg (qui est moins général que celui de Fermat)
    au moins moi je donne des explications ce qui est rarement ton cas
    mais ce n'est pas avec des remarques aussi simplistes
    que tes affirmations ont une quelconque valeur..

  24. #54
    Médiat

    Re : Si Fermat avait la solution..

    suppose que cette solution soit vraie:
    x0^3 + y0^3 = z0^3,
    il existe deux cas possibles
    1)
    dans N =3 avec solutrion aussi dans N=6 et 2:

    x0,y0 et z0 sont trois entiers non nuls premiers entre eux ;tel que x0<y0 <z0 ou y0<x0 <z0
    alors x0,y0 z0 sont trois carrés parfaits
    D'où sort cette affirmation ?

    Sans réponse lisible et un minimum mathématique, je crois que je vais "faire dans l'ignorance", ou bien encore "faire exprès", pour employer un vocabulaire que tu ferais mieux d'éviter si tu veux que ceux qui essayent de t'aider un peu ne se lassent pas trop vite.
    Je suis Charlie.
    J'affirme péremptoirement que toute affirmation péremptoire est fausse

  25. #55
    prgasp77

    Re : Si Fermat avait la solution..

    leg ... Personne ne dit ici que ce que tu écris est faux. Nous sommes juste dérangés dans la comprehension de tes écrits par leur manque de rigueur. Je ne sais pas si tu as déjà codé, mais lire une démonstration non rigoureuse c'est un peu comme lire de l'assembleur sans commentaires ...

    Au lieu de répondre à chaque question (de Médiat entre autres, qui ne te cherche pas misère mais prie depuis plusieurs heures pour que tu comprennes que ce manque de rigueur est pénible), prends ton temps pour faire quelque chose d'irréprochable.

    Citation Envoyé par prgasp77 Voir le message
    Quelques conseils :
    > À chaque symbole introduit, précise ce qu'il représente, de quel ensemble il est élément, s'il est connu/inconnu, fixé/variable etc.
    > Précise à chaque fois les théorème utilisés
    > Explique avant chaque équation ce que tu tentes de faire.
    > Numérote tes équations pour y faire allusion plus tard.
    > Fais des pauses dans ton raisonnement pour résumer ce qui a été démontré jusqu'ici.

    Bonne chance.
    --Yankel Scialom

  26. #56
    leg

    Re : Si Fermat avait la solution..

    bonjour à tous
    prgasp77, tu as entièrement raison, dans tes propos mais j'ai parfaitement compris que Médiat ne me comprend pas et qu'il ne me cherche pas des poux dans la tête;
    mais je suis obligé d'admettre qu'il pense que je confond racine cubique et racine carrée ce qui est idiot!
    jahlucine m'a déjà posé cette question je ne parle que de racine carrée;
    donc que vient faire cette question x0,y0 et z0 de Médiat qui sont des racines cubiques et où il me demande de démontrer que ce sont des racines carrées égale à p²-q²,2pq et p²+q² (ce qui est un peu du temps à perdre, et de l'incompréhension de mes explications)

    petit rappel cette question à déjà fait l'objet d'une mise au point pare zinia sur le sujet de Gaétan au mois d'aout,("nouvelle démo du GTF") post #79,120 et ma réponse ")
    ce qu'il a parfaitement compris, et bien précisé que l'on pouvait effectivement utiliser cette relation pour les équations de Fermat sans perte de généralité ,
    de plus il m'avait demandé ok, mais comment les reconnaîtres , et je lui est répondu...
    désolé mais erik, n'a toujours pas l'air de comprendre que l'on peut extraire 3 racines carrées à trois entiers à la puissance N=3, y compris si ces trois entiers était une équation de Fermat x^3 + y^3 = z^3 avec x,y et z racine cubiques et non racines carrées s'ecrivant sous cette forme p²-q² = x' 2pq = y' et p² +q² = z' il est pourtant clair que x' > x , y' >y et z' >z du simple fait que dans ce triplet de racines carrée:
    x' contient (x et "une part de x") etc pour y' et z'.
    la suite à tout à l'heure

  27. #57
    leg

    Re : Si Fermat avait la solution..

    Citation Envoyé par Médiat Voir le message
    D'où sort cette affirmation ?

    Sans réponse lisible et un minimum mathématique, je crois que je vais "faire dans l'ignorance", ou bien encore "faire exprès", pour employer un vocabulaire que tu ferais mieux d'éviter si tu veux que ceux qui essayent de t'aider un peu ne se lassent pas trop vite.
    bonjour Médiat
    alors x0,y0 z0 sont trois carrés parfaits
    tu me demandes d'où sort cette affirmation
    nous somme bien dans le cas de la supposition d'accord ?
    donc x0^3 + y0^3 = z0^3,est vrai

    il existe deux cas possibles
    1)
    dans N =3 avec solutrion aussi dans N=6 et 2:
    est ce que tu peux concevoir qu'il existerait par obligation dans ce cas, trois carrés tel que:
    (x²)^3 + (y²)^3 = (z²)^3
    et où les racines carrés sont trois cubes
    paramétré par p et q
    tel que p²-q² =(x^3) , 2pq =(y^3) , p²+q² = z^3
    ce qui donne bien x^6 + y^6 =z^6 où
    x0 = x², y0 = y² et z0 = z² ,trois entiers naturels qui sont trois carrés parfaits
    ce qui donne bien aussi cette égalité:
    (x^3)² + (y^3)² = (z^3)²
    correspondant à l'écriture de la formule des triplets Pythagoriciens
    (p²-q²)² + (2pq)² = (p²+q²)² = z0^3 ou z^6 ou (z^3)²

    suite à venir

  28. #58
    leg

    Re : Si Fermat avait la solution..

    suite #57
    le couple de paramètres p et q dans ce cas sont deux entiers naturel tel qu'ils ont été définis
    puisque l'on a une contradiction le couple de paramétres p et q ne peut alors exiter !

    cette solution ne pouvant pas exister par contradiction,

    elle implique par obligation , la non existance d'un couple de paraméttre p' et q' qui serait alors :soit réels soit réels algébriques !

    le contraire ferait croire qu'il existerait un couplet de paramétres réel qui me donne un triplet de cubes, avec cette solution impossible dans N=3 et 6 ainsi que dans N=2
    mais si on suppose que cette solution et possible avec un tel couple de paramèttre p' et q' la démo d'Euler est fausse ,ainsi que celle de A.willes, donc le théorème de Fermat! non ?

    donc mon travail repose justement à démontrer l'absence d'un tel couple de paraméttrage (question que m'avait posé zinia comment reconnaître ....) c'est à dire deux réels ne peuvent exister pour paraméttrer une solution impossible dans un premier temps : dans les puissances premières ce qui est obligatoire

    pourquoi :
    ce que Fermat avait compris , on passe d'abord par les puissances paires pour revenir au puissances premières correspondantes
    car bien évidement pour les puissances premières, le couple de paramétrage p et q sont forcément deux entiers afin de former un triplet primitif de racines carrés

    si le couple d'entiers n'existe pas, du fait que la solution serait impossible par contradiction , elle implique aussi l'absence d'un couple de réels p' et q'

    c'est exactement pareil que si vous dites x² , y² et z n'existe pas donc (x²)² + (y²)² = z² n'existe donc pas de soltuion dans N=4
    avec bien évidement (x²) , (y²) , z : racine carrés

    par conséquent il n'existe pas de couple de paraméttres p et q entiers!

    et je vous dit:
    faux il existe un couple p' et q' réel
    ce à quoi vous me dites : leg

  29. #59
    leg

    Re : Si Fermat avait la solution..

    lireour les puissances pair et non première
    dans cette phrase
    car bien évidement pour les puissances premières,
    désolé

  30. #60
    leg

    Re : Si Fermat avait la solution..

    un autre point de détail:
    si il existe une solution dans la puissance N=4, ou 6, 3 ou uniquement dans N=3
    paramétrer par la formule des triplets
    je pourrai trés facilement en avoir une infinité
    il me suffit pour cela d'utiliser un facteur commun k
    mais il est bien entendu que ce facteur K ne peut être un entier quelconque! c'est une racine carrée d'un entier à la puissnace N concernée, soit :
    pour N =4 , k = un carré parfait

    pour N =6 , k = un cube
    pour N =3 , k = un réel algébrique c'est à dire sqrt de a^3, avec a entier naturel non nul et non carré

Page 2 sur 3 PremièrePremière 2 DernièreDernière

Discussions similaires

  1. Dingue! Einstein avait encore raison!
    Par invitecb70ab37 dans le forum Physique
    Réponses: 43
    Dernier message: 05/07/2004, 17h39